Concours général maths 2021

Bonjour

Pouvez-vous mettre en pdf le sujet du CG maths 2021 qui s’est déroulé aujourd’hui ?

Merci.
«1

Réponses

  • En cherchant sur Twitter, je suis tombé sur le lien lien CG
  • Le même en tout beau tout propre.

    Pierre.
  • La partie proba a l'air dure ::o Je crois que je n'ai rien compris, je ne saurai même pas faire une seule question de la partie probabilité.

    Le problème $1$ a l'air relativement simple. J'ai coincé un peu sur 2 et 3 mais après c'est limpide, je trouvais l'idée directement.

    Question $1$ : l'exponentielle est à valeurs strictement positives.

    Question $2$ :
    Il existe $N \geq 2$ entier tel que $\exp(u_N) \leq e$ soit $\dfrac{\exp(u_N)}{N+1} \leq \dfrac{e}{N+1} $. Ainsi par suite :

    $0 < u_{N+2} \leq \dfrac{\exp(e/ (N+1))}{(N+1)^2}$ et par itération on a $\forall k \geq N \ \ u_{N+k}$ majorée par une suite qui converge vers $0$.

    Question $3$ :

    D'après la question précédente, par contraposée, $\forall N \geq 2 \ \ u_N >1$ donc si $u_n$ converge on a : $l \geq 1$.

    Donc $u$ n'est pas majorée. Montrons qu'elle est croissante. $u_{n+1} /u_n=\dfrac{(n+1)\exp(u_{n+1}-u_n)}{n}$. Le résultat s'obtient par récurrence immédiate.

    Question $4$ :
    $u_2= e/2 \approx 1,4$
    $u_3 \approx 1,35$
    $u_4 \approx 0,96<1$

    Donc d'après la question $2$ la suite $(u_n)$ converge vers $0$.

    Question $5a$ :
    Par récurrence sur $n$. Soit $x<y$. Si $n=0$, on a $u_n(x)=x <u_n(y)=y$
    Si $P_n$ est vraie alors $u_n(x) <u_n(y)$. Or $u_{n+1} (x)=\dfrac{ \exp u_n(x) }{n+1} < \dfrac{ \exp u_n(y) }{n+1}$ car $x<y \implies \exp(x)<\exp(y)$.

    Question $5b$ :
    $\forall y \in ]-\infty,x]$ on a $u_n(y) \leq u_n(x)$ et le théorème des gendarmes permet de conclure.

    Question $6.a$ :
    Posons $g(x)=e^x-x(x+1)$ on a $g'(x)=e^x-(x+1)-x=e^x-2x-1$
    $g'(x)>0$ si et seulement si $x >\ln(2x+1)$ soit $x>2$.
    Donc $g$ est strictement croissante sur $[2,+\infty[$ or $g(2)=e^2-6 >0$

    Question $6.b$ :
    S'il existe un rang $N \geq 1$ tel que $u_N \geq N+1$ alors $u_{N+1} = \dfrac{\exp (u_N)}{N+1} > u_N \geq N+1$ d'après la question précédente.
    Donc $\forall n \geq N$ on a $u_n \geq N+1$ et par comparaison $u$ diverge vers $+\infty$.

    Question $6.c$ :
    $u_0=1$, $u_1=e \geq 2=1+1$ la question précédente permet de conclure immédiatement.

    Question $7$ :
    $\forall y \in [x,+\infty[$ on a $u_n(y) \geq u_n(x)$. Or $u_n(x)$ diverge vers $+\infty$ donc par comparaison $u_n(y)$ aussi.
  • Dès la question 2), c'est une horreur. Arrête, retourne, faire tes DL, ça te sera plus profitable.
  • J'ai corrigé la coquille.
    La question $2$ est plus dure qu'elle n'y parait.
  • Non, ça reste fantaisiste. "Par itération" est très vague donc si tu fais une récurrence, fais la proprement. Ton $u_{N+k}$ majoré, on voit pas trop par quoi, on ne sait pas si $N$ est fixé ou pas, idem pour k...
  • Mais est-ce que ce n'est pas fantaisiste de parler de "la'' variable aléatoire D_n et où cette variable aléatoire est-elle définie d'ailleurs... B-)
    Merci pour le sujet!
  • Oui Alexique une recurrence régle l'affaire.
    J'essaierai de terminer le problème 1.
  • Pour moi, tes réponses aux questions 2),3),6a),6b) sont incorrectes. Donc tu peux continuer tant que tu veux, faire tout le sujet et faire comme s'il n'y avait pas de problème bien sûr, c'est ton droit.
  • Bonjour.

    OShine, OShine, OShine. Ce que tu écris est du niveau des pires trolls de ce forum. Tu peux d'ailleurs leur donner la main pour le fait de démontrer sans utiliser les hypothèses dans le corps de la preuve. Pour la question 2, $N\ge 2$ ne sert pas; on aurait pu prendre $N=0$ ou $N=1$, de la même façon. Pourquoi $U_n\le 1$ ? On aurait pu prendre $U_n\le 0.5$ ou $U_n\le 75000$, de la même façon. 2 détails qui indiquent que ta démonstration est fausse.

    De plus, si $U_N\le 1$, alors $U_{N+1}\le \frac{e}{N+1}$, puis $U_{N+2}\le \frac{exp(\frac{e}{N+1})}{N+2}$. D'où provient ton (N+1)^2 au dénominateur ?

    Dans la même veine, dans la question 3, par quel miracle Un+1 et Un peuvent se trouver au même niveau dans la même exponentielle alors que, a priori, il y aura toujours un niveau d'exponentielle entre les deux ?

    Enfin, dans la question 2, le terme de droite de ton encadrement de UN+2 est fixe. e est fixe. N est fixe. Donc le terme de droite est juste un nombre non nul, fixe. Et UN+2 fait ce qu'il veut. Il convergera que s'il a envie. Ou pas du tout.

    C'est nul. Aussi bien d'un point de vue de l'intuition, que de la besogne. Change de métier.
    Ce site est fatigant. Les gens modifient sans cesse leurs messages passés, et on ne comprend plus rien à la discussion. Je suis nostalgique du temps où, si on postait une bêtise, on devait l'assumer. Et si on cite le passage pour l'ancrer, l'administrateur supprime en disant qu'on n'a pas besoin de recopier le message passé.
  • Cette épreuve est d'un niveau relevé.

    Petitlutinmalicieux
    Alexique
    Ok je revois ça de A à Z, je me suis mélangé les pinceaux j'ai fait n'importe quoi :-(
  • Je ne trouve pas à quoi sert l'hypothèse $N \geq 2$. Comment utiliser une hypothèse alors qu'on ne comprend pas à quoi elle sert ?
  • Au problème 1, jolie suite réelle $u_n$ régie par une récurrence $u_{n+1}=f(n,u_n)$, où apparaît une constante $\delta $ telle que $u_n \rightarrow 0$ si $u_0< \delta $ et $u_n \rightarrow +\infty $ si $u_0 \ge \delta $. J'avais déjà cet énoncé depuis longtemps dans mes archives de problèmes de colles pour Math. sup. J'en joins un énoncé et un corrigé que j'avais rédigés en 2007, mais le problème date de bien avant ; je vais essayer de retrouver une référence Un mérite de l'énoncé du Concours général c'est de proposer un argument élémentaire pour avoir la limite de la suite lorsque $u_0= \delta $, ce qui n'est pas le cas de mon corrigé.
    Les membres du jury connaissent l'état de l'enseignement secondaire actuel, ce qui explique sans doute la trivialité de la question 1, probablement conçue pour ne pas « désespérer Billancourt ».
    Le problème 3 propose des notions intéressantes, que je ne connaissais pas. Rien à dire pour l'instant sur le problème 2, que je n'ai pas regardé de près.
    Peut-être peut-on regretter qu'il y ait deux problèmes d'Analyse et un de Probabilités, sans Arithmétique, ni Géométrie, ni Combinatoire, ce qui éloigne des Olympiades internationales, où il n'y a pas, sauf erreur, de Calcul des probabilités. Mais on me dira que le Concours général a sa spécificité, et le jury est composé de professeurs compétents, qui savent ce qu'ils font.
    Bien amicalement à PierreB.
    Bonne soirée
    Fr. Ch.
  • Il y a eu de l'arithmétique dans les sujets précédents de CG, mais cette année l'arithmétique est traitée uniquement en maths expertes. Or le CG n'est pas réservé uniquement aux élèves ayant pris cette option.
  • OShine: $\text{e}>2$.
  • Le problème 2 m'a l'air le plus difficile et de loin. C'est des probas bien théoriques et ça m'a l'air plus dur que certains concours de prépa comme CCPINP.

    Chaurien merci.

    S'il existe $N \geq 2$ tel que $u_N \leq 1$.
    Soit $n \geq N$. $u_{n+1}=\dfrac{e^{u_n}}{n+1} < \dfrac{e}{n+1} \leq \dfrac{e}{N+1} \leq \dfrac{e}{3} <1$

    Je ne comprends pas la suite de ton corrigé de la question.
  • OShine a écrit:
    Je ne trouve pas à quoi sert l'hypothèse $N \geq 2$.

    Peut-être parce que si $N\ge 2$ alors $N+1\ge 3$. Et comme "e" = 2.71828 < 3, en divisant par 3 (ou au moins 3), on reste en dessous de 1 au rang suivant. Peut-être même que la suite devient définitivement décroissante, mais positive.

    Image = représentation de la suite, avec $U_0$ de 0 à 1.3 avec un pas de 0.1.119378
    Ce site est fatigant. Les gens modifient sans cesse leurs messages passés, et on ne comprend plus rien à la discussion. Je suis nostalgique du temps où, si on postait une bêtise, on devait l'assumer. Et si on cite le passage pour l'ancrer, l'administrateur supprime en disant qu'on n'a pas besoin de recopier le message passé.
  • D'accord merci.

    J'ai montré $\forall n \geq N \geq 2, \ \ u_{n+1} <1$

    Je n'arrive pas à en déduire que $(u_n)$ converge vers $0$.
  • Je suis certain que je n'ai pas inventé le problème 1, et qu'il provient d'un énoncé d'oral de concours. Malheureusement, je n'en ai pas retrouvé l'origine. Mais j'ai retrouvé une feuille d'exercices que j'avais donnée à mes élèves de Math. spé en 1993-94, où il figure en n° 21.
    Bonne soirée.
    Fr. Ch.
  • Chaurien tu vas trop vite dans ton corrigé pour la question $1$.

    Je ne comprends pas le passage encadré en rouge.119384
    1.png 102.4K
  • Chaurien a écrit:
    où il n'y a pas, sauf erreur, de Calcul des probabilités.

    Tu as sans doute raison pour l'essentiel, cependant, pour te taquiner, le 2e problème de l'olympiade de 1981 est clairement un calcul d'espérance (qui se fait d'ailleurs très bien avec la formule que tu aimes bien).
  • Comment jugez-vous la difficulté de chacun des 3 problèmes les profs expérimentés ? Facile, moyen, difficile, très difficile ?
  • Bonsoir,

    Ça n'a pas d'importance, OShine, l'essentiel est qu'ils soient intéressants.
    Comment faut il t'asséner la signification du mot "concours" ?

    Cordialement,

    Rescassol
  • @ OShine
    Je n'avais pas relu, mais c'est visiblement faux, puisque ceci signifierait que $u_{n-1}<0$ !
    Bravo pour ta vigilance. Je te laisse le plaisir de corriger.
    Bonne soirée.
    Fr. Ch.
  • @ alea
    Oui, tout calcul de moyenne, ou même de somme, peut être ainsi interprété.
    J'ai posé moi-même de tels problèmes dans des classes où les probabilités n'étaient pas au programme.
    On peut évoquer aussi le problème n° 1 de 1987 (Cuba).
    Mais rien qui approche le beau problème n° 2 du Concours général de cette année
    Bonne soirée.
    Fr. Ch..
  • Rescassol c'était pour avoir un ordre d'idée avant de chercher les exercices. Si c'est vraiment difficile, je préfère éviter.

    Ok Chaurien j'ai trouvé la solution finalement. Ces questions $2$ et $3$ ne sont pas insurmontables finalement.

    Le problème $2$ a l'air joli mais ardu.

    Question $2$ :
    On a $\forall n \geq N \geq 2$ : $u_{n+1} \leq 1$

    Donc $\forall n \geq N+1 \geq 3$ : $u_{n} \leq 1$ ainsi $e^{u_n} \leq e$ et enfin $u_{n+1} \leq \dfrac{e}{n+1}$

    Ainsi on a montré $\boxed{\exists N \geq 2 \ \forall n \geq N+1 \ \ \ \ 0<u_{n+1} \leq \dfrac{e}{n+1}}$

    Le théorème d'encadrement permet de conclure puisque $\lim\limits_{n \rightarrow +\infty} \ln(n+1)=+\infty$

    Et la question $3$ :
    Si la suite ne converge pas vers $0$ alors $\forall n \geq 2 \ \ u_n >1$. Donc $\forall n \geq 1 \ \ u_{n+1} >1 \implies \dfrac{e^{ u_n}}{n+1} >1 \implies u_n > \ln(n+1)$

    On conclut avec le théorème d'encadrement.

    Je vais maintenant chercher à corriger mes réponses aux questions $6a$ et $6b$ fausses.
  • @Chaurien : Si tu retrouves une référence pour le pb. I, oui, ça m'intéresse.

    Concernant le contenu, d'après les consignes officielles, le programme de référence pour le CG 2021 est celui de spécialité mathématique, mais pas celui des maths expertes. De fait, tu élimines l'arithmétique.
    La géométrie et la combinatoire sont également très maltraitées actuellement, même si des raisonnements combinatoires sont utiles au pb. II.
    Il faut aussi tenir compte de ce que certains concepts comme les intégrales ne font partie que du programme de toute fin d'année (je n'ai pas le B.O. mais c'est marqué), soit donc clairement après l'épreuve. Ainsi, pas d'intégrale non plus, il faut être fair-play.

    Il faudra voir si cette consigne reste vraie pour les années à venir ou si on peut considérer que des élèves qui souhaiteraient présenter le CG de math ont quand même au moins pris la spé maths expertes. Mais ça, c'est le Ministère qui décide.

    Enfin, quoi qu'on en pense, le CG n'a aucune vocation à ressembler aux olympiades et ne sert plus du tout d'outil de sélection pour les OIM par exemple. Pour cela, il y a la P.O.F.M. qui travaille efficacement (surtout avec les "moyens" dont elle dispose).

    Donc, oui, l'idée est de trouver des trucs intéressants (même si, les goûts et les couleurs...), variés et abordables par des élèves de Tle qui n'ont pas fait le programme de prépa en avance. Que chacun puisse travailler 5h à son rythme, avec un certain nombre de questions discriminantes pour pouvoir établir un palmarès.

    A titre d'exemple, d'après les échos que j'en ai, il semblerait que, parmi les olympiques les plus avancés, personne n'ait fini le sujet mais que certains ont les pb. I et II complets et la première partie (ou un peu plus) du III. Le pb. III est sans doute moins difficile que le pb. II mais il est placé en III.... Il apparaissait en effet inconcevable de ne pas mettre "la loi du milieu"... au milieu. Maintenant, rien n'interdit que des candidats non olympiques puissent faire mieux.

    Pierre.
  • OShine, ta solution de la question 2) est fausse, tu confonds $\exists$ et $\forall$.

    Supposons donc qu'il existe $N\geqslant 2$ tel que $u_N\leqslant 1$. On montre par récurrence que $u_n\leqslant 1$ pour tout $n\geqslant N$. En effet, si $u_n\leqslant 1$ alors $u_{n+1}\leqslant \dfrac{e}{n+1}\leqslant \dfrac{e}{3}\leqslant 1$.

    On a alors $\forall n\geqslant N+1$ : $u_n=\dfrac{e^{u_{n-1}}}{n}\leqslant \dfrac{e}{n}$ donc $u_n\to 0$.
  • JLT ok merci pour cette précision.

    D'après les rapports du jury la combinatoire est maltraitée même dans les concours d'ingénieur.
    En même temps, la combinatoire est un domaine qui peut vite être très compliqué.

    Oui je confirme le III m'a l'air abordable à la différence du $2$ ou je coince dès la question $2$.

    Question $6.a$ :
    Posons $\forall x \in [2,+\infty[ \ g(x)=e^x-x(x+1)$
    On a $g'(x)=e^x-(x+1)-x=e^x-2x-1$
    Or $g'(x) >0 \Leftrightarrow e^x >2x+1$
    Posons $h(x)=e^x-2x-1$. On a $h'(x)=e^x-2$ et $h'(x) >0 \Leftrightarrow x > \ln(2) $
    $h$ est strictement croissante sur $[2,+\infty[$ et $h(2)=e^2-4-1>0$ donc $h$ est positive.
    Ce qui montre que $\forall x \in [2,+\infty[ \ g'(x) >0$
    Donc $g$ est strictement croissante sur $ [2,+\infty[$ et $g(2)=e^2-6>0$
    La fonction $g$ est donc croissante sur $ [2,+\infty[$.

    Question $6.b$ :
    Supposons qu'il existe un rang $N \geq 1$ tel que $u_N \geq N+1$.
    Alors $e^{u_N} \geq e^{N+1}$ et $u_{N+1} \geq \dfrac{e^{N+1}}{N+1}$. Par récurrence :
    $\forall n \geq 2 \ \ u_{n} \geq \dfrac{e^{n}}{n}$
    Par croissances comparées, $\boxed{\lim\limits_{n \rightarrow +\infty} \dfrac{e^{n}}{n}=+\infty \implies \lim\limits_{n \rightarrow +\infty} u_n=+\infty}$

    Question $8.a$ : le niveau semble s'élever avec cette question. Elle demande une prise d'initiative et me semble bien difficile pour un élève de terminale. J'ai direct eu l'idée car elle me rappelle la démonstration du théorème des valeurs intermédiaires.

    Montrons que les suites $(a_n)$ et $(b_n)$ sont adjacentes.
    On a $a_{n+1}-b_{n+1}= \begin{cases}
    \dfrac{a_n-b_n}{2} \ \text{si} \ \ (a_n+b_n)/2 \in E_0 \ \\
    \dfrac{a_n-b_n}{2} \ \text{sinon} \\
    \end{cases}$

    Dans tous les cas $a_{n+1}-b_{n+1}= \dfrac{1}{2} (a_n-b_n)$ donc $\forall n \in \N \ a_n-b_n= \dfrac{1}{2^n} (a_0-b_0)$

    Finalement $\boxed{\forall n \in \N \ a_n-b_n= \dfrac{-1}{2^n}}$

    Donc la suite $(a_n-b_n)$ converge vers $0$. Remarquons aussi que la suite $(b_n-a_n)$ est positive, remarque important pour la suite.

    Par ailleurs, $a_{n+1}-a_{n}= \begin{cases}
    \dfrac{b_n-a_n}{2} \ \text{si} \ \ (a_n+b_n)/2 \in E_0 \ \\
    0\ \text{sinon} \\
    \end{cases}$

    Donc la suite $(a_n)$ est croissante.

    En outre, $b_{n+1}-b_{n}= \begin{cases}
    0 \ \text{si} \ \ (a_n+b_n)/2 \in E_0 \ \\
    \dfrac{a_n-b_n}{2} \ \text{sinon} \\
    \end{cases}$

    Donc la suite $(b_n)$ est décroissante.

    Les suites $(a_n)$ et $(b_n)$ sont donc adjacentes, elles convergent vers une même limite.

    Question $8.b$ : non achevée
    D'après la question précédente, on a $\boxed{\forall n \in \N \ a_n \leq \delta \leq b_n}$

    Je réfléchis encore à cette question, j'espère la trouver car j'ai des idées.
  • Je coince à la $8.b$. Soit $y \in ]-\infty,\delta[$ alors d'après la question $5.a$ on a $u(y) \in ]-\infty,u(\delta)[$.

    Je ne vois pas comment utiliser les suites $a_n$, $b_n$...
  • Ton $h$ de la question 6a) ne sert à rien, c'est juste $g'$ que tu redérives en $g''$ ce qui est du bon sens, vu que tu devais comparer $\exp$ à un polynôme de degré 2, il faut dériver 2 fois... C'est pas faute d'avoir mangé des DL récemment, ça ne t'as encore pas profité visiblement.

    Fais un dessin. $\delta$ fait la césure entre $E_0$ et $E_{\infty}$ et est la limite des $(a_n)_n$, suite croissante d'éléments de $E_0$...
  • C'est vrai que le dessin aide ! C'est une question de synthèse qui utilise beaucoup de questions précédentes, je l'ai résolue à moitié, il y a un détail que je n'arrive pas à montrer.

    Montrons que $\forall n \in \N \ a_n \in E_0$ par récurrence sur $n$. Notons $P_n$ la propriété.
    Au rang $n=0$ : $a_0=0 \in E_0$
    Supposons $P_n$ vraie.
    Si $\dfrac{a_n+b_n}{2} \in E_0$ alors $a_{n+1}=\dfrac{a_n+b_n}{2} \in E_0$
    Sinon $a_{n+1}=a_n \in E_0$
    Dans tous les cas $a_{n+1} \in E_0$

    On a montré $\boxed{\forall n \in \N \ a_n \in E_0}$

    D'après la question $5b$, $\forall n \in \N \ ]-\infty,a_n] \subset E_0$. Je suis proche mais je ne vois pas comment en déduire l'inclusion souhaitée :-S

    Montrons que $\forall n \in \N \ b_n \in E_{\infty}$ par récurrence sur $n$. Notons $Q_n$ la propriété.
    Au rang $n=0$ : $b_0=1 \in E_{\infty}$ d'après la question $6c$.
    Supposons $Q_n$ vraie.
    Si $\dfrac{a_n+b_n}{2} \in E_0$ alors $b_{n+1}=b_n \in E_0$
    Sinon $b_{n+1}=\dfrac{a_n+b_n}{2} \notin E_0 $. D'après la question $3$, on en déduit que $b_{n+1} \in E_{\infty}$

    On a montré $\boxed{\forall n \in \N \ b_n \in E_{\infty}}$

    D'après la question $7$, $\forall n \in \N \ [b_n,+\infty[ \subset E_{\infty}$. Même blocage :-S
  • Est-ce que tu as utilisé $\delta=\lim a_n=\lim b_n$ ?
  • J'ai pensé à l'utiliser mais je n'ai pas abouti.

    On a $\forall n \in \N \ ]-\infty,a_n] \subset E_0$.

    On a le droit de passer à la limite dans un intervalle ? Pourquoi la borne est ouverte en $\delta$ ? Ce sont les 2 points qui m'ont empêché de conclure hier.
  • Si $(a_n)$ est une suite croissante qui converge vers un réel $\delta$, alors

    $$\,\,]\!-\infty,\delta[\subset \bigcup_{n\in\N} \,\,]\!-\infty,a_n].$$

    Exercice : le démontrer.
  • Ou alors passer à la définition par des $\varepsilon$ en choisissant bien ce dernier.
  • D'accord, merci JLT je vois le principe.

    Soit $y \notin \bigcup_{n\in\N} \,\,]\!-\infty,a_n]$

    Alors $\forall n \in \N \ \ y \notin \ ]-\infty,a_n]$ Donc $\forall n \in \N \ \ y>a_n$. Par passage à la limite $y \geq \delta$.
    Ainsi, $y \in [\delta,+\infty[$.

    Par contraposée on obtient le résultat.

    Nahar je ne trouve pas le $\varepsilon$.

    Soit $\varepsilon >0$. Il existe un rang $N \in \N$ tel que $n \geq N \implies |a_n-\delta| \leq \varepsilon$

    Soit $n \geq N \implies \delta-\varepsilon \leq a_n \leq \delta+\varepsilon$
  • Soit $x\in ]-\infty,\delta[$.
    Il existe $N\in \mathbb N: (\forall n\ge N), x\ <a_n \le \delta$ o(n prend $\varepsilon = \delta-x$).
    On a: $x\in ]-\infty,a_N[$.
  • Ok merci.

    Je vais faire une pause et essayer de résoudre les $3$ dernières question du problème.
  • La question $9$ me pose des problèmes.

    J'arrive à montrer que la propriété est vraie au rang $2$ mais je n'arrive pas à faire la récurrence.

    On a $u_0(c_2)=c_2$, $u_1(c_2)=e^{c_2}=\ln 2$
    $u_2(c_2)=1$
    D'après la question $2$, on a $c_2 \in E_0$.

    Pour l'hérédité ça me semble bien compliqué...
  • Ben essaye au rang 3. Et si tu ne vois toujours pas, essaye au rang 4.
  • Oui c'est vrai. Faire le cas $l=3$ m'a permis de comprendre d'où proviennent tous ces $\ln$ imbriqués. Mais je n'ai pas réussi à faire de lien entre $c_l$ et $c_{l+1}$ pour trouver l'hérédité :-S

    Cas $l=3$ :
    $c_3=\ln (\ln (2 \ln 3)))$
    $u_0(c_3)=c_3$
    $u_1(c_3)=\ln (2 \ln 3))$
    $u_2(c_3)=\ln 3$
    $u_3(c_3)=3/2=1,5$
    $u_4(c_3)=\dfrac{e^{1,5}}{3} \approx 1,5$
    $u_5(c_3) \approx 1,12$

    $c_4=\ln (\ln (2 \ln (3 \ln 4 ))))$

    On remarque qu'à partir d'un certain rang on aura $u_n(c_l) \leq 1$...

    Hérédité :
    Supposons la propriété vraie au rang $l$ fixé avec $l \geq 2$.
    On a donc $c_l \in E_0$

    J'ai remarqué que $u_l(c_l)=\dfrac{l}{l-1}$ et $u_l(c_{l+1})=\ln (l+1)$
  • La question ne se résout pas par récurrence (ou du moins pas avec la récurrence que tu crois).
  • Ok merci du coup je vais chercher d'autres idée j'étais obnubilé par la récurrence.
  • $u_2(c_3)=\ln 3$ donc $u_3(c_3)=3/2=1,5$??
    Relis la définition : $u_{n+1}=\dfrac {e^{u_n}}{n+1}$.
    Regarde bien les $n+1$
  • Ok merci Nahar une coquille en effet. Je reprends :

    $u_0(c_3)=\ln (\ln 2 \ln(3)))$
    $u_1(c_3)=\ln (2 \ln 3=$
    $u_2(c_3)= \ln (3)$
    $u_3(c_3)= \dfrac{3}{3}=1$

    Soit $l \geq 2$. On a $u_l (c_l)=1 \leq 1$ et d'après la question $2$, on en déduit que $c_l \in E_0$

    Je me suis compliqué la vie pour rien c'était très simple.
  • Bientôt la fin.

    Question $10$ :
    La suite $(c_l)$ est clairement croissante car la fonction $\ln$ est strictement croissante et $\forall l \geq 2 \ \ l \ln(l+1) > l$

    En effet, $\forall l \geq 2 \ \ \ln(1+l) >1$

    Il suffit de montrer que $(c_l)$ est majorée. Et là je coince même si je pense qu'il faut utiliser que $c_l \in E_0$ mais je ne vois pas comment.
    Je ne vois pas le lien avec les suites $(a_n)$ et $(b_n)$.
  • OS a écrit:
    Soit $l \geq 2$. On a $u_l (c_l)=1 \leq 1$
    Pas démontré.
    OS a écrit:
    Il suffit de montrer que $(c_l)$ est majorée. Et là je coince même si je pense qu'il faut utiliser que $c_l \in E_0$ mais je ne vois pas comment$.
    Si seulement $E_0$ était majoré 8-)
  • En fait c'est juste que c'est galère à écrire avec tous ces $\ln$ et toutes ces parenthèses. Mais je dirais que $u_l(c_l)=u (u_{l-1} (c_l))$ etc...
    A chaque fois qu'on applique $u$ il y a un logarithme qui disparait.

    Je ne vois pas pourquoi $E_0$ serait majoré, j'ai relu toutes les questions.
  • Ben oui, mais du coup, si tu ne fais pas cet effort, on peut pas dire que tu aies démontré quelque chose. Dire "ça se voit bien que...", ça ne fait pas une démo.

    Ben c'est triste que tu n'aies toujours pas compris qu'on avait $E_0$ plus petit que $\delta$ plus petit que $E_{\infty}$ en gros...
    Par la 5b), par exemple, c'est très facile de montrer que $E_0$ est majoré.
Connectez-vous ou Inscrivez-vous pour répondre.